Measurable Functions .... Lindstrom, Proposition 7.3.7 .... ....

In summary: X | g(x) \lt r - q \}##This completes the proof.In summary, Peter's problem has been resolved and he is now able to understand the proof of Proposition 7.3.7.
  • #1
Math Amateur
Gold Member
MHB
3,990
48
TL;DR Summary
I need help with Tom Lindstrom's proof that if f and g are measurable then so is f + g ...
I am reading Tom L. Lindstrom's book: Spaces: An Introduction to Real Analysis ... and I am focused on Chapter 7: Measure and Integration ...

I need help with the proof of Proposition 7.3.7 ...

Proposition 7.3.7 and its proof read as follows:
Lindstrom - 1 - Proposition 7.3.7 ... PART 1 .png

Lindstrom - 2 - Proposition 7.3.7 ... PART 2 .png
In the above proof by Lindstrom we read the following:

" ... ... ##(f + g)^{ -1} ( [ - \infty , r ) ) = \{ x \in X | (f + g) \lt r \}##

## = \bigcup_{ q \in \mathbb{Q} } ( \{ x \in X | f(x) \lt q \} \cap \{ x \in X | g \lt r - q \} ) ## ... ... "Can someone please demonstrate, formally and rigorously, how/why ...

##\{ x \in X | (f + g) \lt r \} = \bigcup_{ q \in \mathbb{Q} } ( \{ x \in X | f(x) \lt q \} \cap \{ x \in X | g \lt r - q \} )## ... ...
Help will be much appreciated ...

Peter=============================================================================================================Readers of the above post may be assisted by access to Lindstrom's introduction to measurable functions (especially Lindstrom's definition of a measurable function, Definition 7.3.1) ... so I am providing access to the relevant text ... as follows:

Lindstrom - 1 - Section 7.3 ... Measurable Functions ... Part 1 .png

Lindstrom - 2 - Section 7.3 ... Measurable Functions ... Part 2 .png

Lindstrom - 3 - Section 7.3 ... Measurable Functions ... Part 3 .png

Hope that helps ...

Peter
 
Last edited:
Physics news on Phys.org
  • #2
##\subseteq##

Given ##f(x)+g(x)< r##, use density of the rational numbers in the real numbers to choose ##q\in \mathbb{Q}## with ##f(x)< q < r- g(x)##.

##\supseteq##

Is trivial.
 
  • #3
Sorry ... but i don't follow ... not completely anyway ...

Apologies for being a bit slow ...

Peter
 
  • #4
Math Amateur said:
Sorry ... but i don't follow ... not completely anyway ...

Apologies for being a bit slow ...

Peter

Refresh the page ( I made an edit). What don't you understand?
 
  • Like
Likes Math Amateur
  • #5
Hmmm ... we require ##\{ x \in X | (f + g) (x) \lt r \}## to be measurable for all ##r \in \mathbb{R}## ... ... surely this is true because ##(f + g)(x) = f(x) + g(x)## and ##f## and ##g## are measurable ...

So why do we have to invoke the density of the rationals ...

Can you try to make more clear what is going on ...?

Peter

EDIT ... cannot understand what has gone wrong with my Latex code
 
  • #6
Math Amateur said:
Hmmm ... we require ##\{ x \in X | (f + g) (x) \lt r \}## to be measurable for all ##r \in \mathbb{R}## ... ... surely this is true because ##(f + g)(x) = f(x) + g(x)## and ##f## and ##g## are measurable ...

So why do we have to invoke the density of the rationals ...

Can you try to make more clear what is going on ...?

Peter

That's another question than the one you asked!

However, you are exactly trying to prove that ##f+g## is measurable when ##f## and ##g## are. To check measurability, you have to show that (note my superior notation) ##\{f+g < r\}## is a measurable set for all ##r##. The proof then writes ##\{f +g < r\}## as a countable union of measurable sets, to deduce that ##\{f+g< r\}## is measurable. For this, the hypothesis that ##f## and ##g## are measurable is used. This countable aspect is very important: the uncountable union of measurable sets need not be measurable.
 
  • #7
Note however, there is an alternative proof. You can prove that addition ##+: \mathbb{R}^2 \to \mathbb{R}## is measurable (because it is continuous) and you can also quickly prove that composition of measurable functions is measurable. Hence, if ##f,g## are measurable, then so is ##f+g = + \circ (f,g)##.

Also, there seems to be some bugs with the Latex code for me as well. Try to refresh the pages when you are done editing.
 
  • #8
Thanks ... can definitely see the point about countability ... and also that f < q < r - g implies that ... f < q and (intersect) g < r - q is the case ...

Peter
 
  • #9
Math Amateur said:
Thanks ... can definitely see the point about countability ... and also that f < q < r - g implies that ... f < q and (intersect) g < r - q is the case ...

Peter

Then is your problem solved or are there any other issues left?
 
  • #10
Still reflecting on things ... but think issues are resolved ...

Most grateful for your help ...

Thanks again ...

Peter
 
  • Like
Likes member 587159
  • #11
I have been reflecting further on proving that ##f+ g## is measurable when ##f## and ##g## are measurable ... and I need to clarify an issue ... ...

I need to understand exactly what is wrong with the following argument ... something surely is wrong because the proof doesn't involve ranging over ##\mathbb{Q}## ...

... ... ... the argument follows ... ...First note that ##f + g \lt r##

##\Longrightarrow## there exists ##q \in \mathbb{Q}## such that ##f \lt q \lt r - g##

##\Longrightarrow f \lt q## and ##g \lt r - q##

##\Longrightarrow## for all ##x \in X## we have ##f(x) \lt q## and ##g(x) \lt r - q##

Thus ##\{ x \in X \ | \ (f + g) \lt r \} = \{ x \in X \ | \ f(x) \lt q \} \cap \{ x \in X \ | \ g(x) \lt r - q \}##Can someone please clarify what is wrong with the above argument ...
Help will be much appreciated ...

Peter
 
Last edited:
  • #12
Math Amateur said:
First note that ##f + g \lt r##

##\Longrightarrow## there exists ##q \in \mathbb{Q}## such that ##f \lt q \lt r - g##

This isn't right (and I'll admit the given proof glosses over this point a little bit), because ##f## and ##g## are functions. For example, suppose ##f(x)=x## and ##g(x)=-x## and let ##r=1.## Then ##0=f(x)+g(x)<1=r## is true for all ##x##, but there is no ##q## such that ##f(x)<q## for all ##x##.

A little more conceptually: the given proof is saying that for every ##x##, there is some ##q\in\mathbb{Q}## (depending on ##x##) such that ##f(x)<q## and ##g(x)<r-q##. That ##q## may need to depend on ##x## is why a union over all rationals is needed. You are trying to escape the dependence on ##x##.
 
  • Like
Likes Math Amateur
  • #13
Infrared said:
This isn't right (and I'll admit the given proof glosses over this point a little bit), because ##f## and ##g## are functions. For example, suppose ##f(x)=x## and ##g(x)=-x## and let ##r=1.## Then ##0=f(x)+g(x)<1=r## is true for all ##x##, but there is no ##q## such that ##f(x)<q## for all ##x##.

A little more conceptually: the given proof is saying that for every ##x##, there is some ##q\in\mathbb{Q}## (depending on ##x##) such that ##f(x)<q## and ##g(x)<r-q##. That ##q## may need to depend on ##x## is why a union over all rationals is needed. You are trying to escape the dependence on ##x##.


Thanks for the help Infrared ...

I can see ##q## may depend on ##x## ... but cannot see why a union over all \mathbb{Q} resolves this issue ...

... why are we justified in taking a union over all ##\mathbb{Q}## ...

Can you explain further ...

Peter
 
  • #14
Let's formally show that $$\{f+g < r\}= \bigcup_{q \in\mathbb{Q}}\{f < q\} \cap \{g < r-q\}$$
Suppose that ##x \in \{f+g < r\}##. Thus ##f(x) + g(x) < r##, so ##f(x) < r-g(x)##. By density of ##\mathbb{Q}## in ##\mathbb{R}##, we can pick ##q(x) \in \mathbb{Q}## (note the ##x##-dependency) such that ##f(x)< q(x) < r-g(x)##. Then clearly we have ##x \in \{f< q(x)\}\cap \{g < r-q(x)\} \subseteq \bigcup_{q \in \mathbb{Q}} \{f < q\}\cap \{g< r-q\}##. This shows
$$\{f+g < r\}\subseteq \bigcup_{q \in \mathbb{Q}} \{f< q\}\cap \{g < r-q\}$$
Next, suppose ##x \in \bigcup_{q \in \mathbb{Q}} \{f< q\}\cap \{g < r-q\}##. Then there is ##q(x) \in \mathbb{Q}## with ##f(x)< q(x)## and ##g(x) < r-q(x)##. Then ##f(x) + g(x) < q(x) + r-q(x) = r## and thus ##x \in \{f+g < r\}##. This shows
$$\bigcup_{q \in \mathbb{Q}} \{f< q\}\cap \{g < r-q\}\subseteq \{f+g < r\}$$
 
  • Like
Likes Math Amateur
  • #15
Hi Math_QED ...

Thank you for an exceedingly clear and helpful post ...

That resolves all issues ...

Thanks again ...

Peter
 

What is a measurable function?

A measurable function is a function between two measurable spaces that preserves the structure of the spaces. In other words, the pre-image of any measurable set in the output space must be a measurable set in the input space.

What is the significance of Proposition 7.3.7 in Lindstrom's theorem?

Proposition 7.3.7 in Lindstrom's theorem states that if a function is measurable from one measurable space to another, then the composition of that function with any other measurable function is also measurable. This is important because it allows us to prove the measurability of more complex functions by breaking them down into simpler, measurable functions.

How is measurability related to integration?

Measurable functions play a crucial role in integration, as they are the functions that can be integrated over a measurable set. In other words, the integral of a function can only be defined over a measurable set if the function itself is measurable.

What are some examples of measurable functions?

Examples of measurable functions include continuous functions, step functions, and piecewise functions. Any function that can be broken down into simpler, measurable functions is also considered measurable.

Why is the concept of measurability important in mathematics?

Measurability is important in mathematics because it allows us to define and work with integrals, which are essential in many areas of mathematics and science. It also helps us to understand the structure of measurable spaces and how functions interact with them.

Similar threads

Replies
8
Views
1K
Replies
6
Views
2K
Replies
4
Views
1K
Replies
2
Views
2K
Replies
4
Views
2K
  • Topology and Analysis
Replies
3
Views
2K
  • Topology and Analysis
Replies
2
Views
2K
Replies
7
Views
2K
Replies
2
Views
2K
Replies
2
Views
1K
Back
Top